RR - Dermatology 1
A 52-year-old woman is brought to the emergency department with burns from a house fire. Physical exam reveals superficial burns over her entire left arm and partial-thickness and full-thickness burns covering her entire right arm, her anterior right leg and anterior trunk. Which of the following percentages of total body surface area best estimates the extent of her burns? 18% 36% 45% 9%
36% The extent of burn size in this patient is 36% total body surface area. A thorough and accurate estimation of burn size is essential to guide therapy and to determine when to transfer a patient to a burn center. The extent of burns is expressed as the total percentage of body surface area. Superficial burns are not included in the burn assessment. For adult assessment, the most expeditious method to estimate total percentage of body surface area is the Rule of Nines. This method only takes into account partial-thickness and full-thickness burns. Each leg represents 18% total percentage of body surface area; each arm represents 9% total percentage of body surface area; the anterior and posterior trunk each represent 18% total percentage of body surface area; and the head represents 9% total percentage of body surface area. This patient has partial-thickness and full-thickness burns covering her entire right arm, her anterior right leg and anterior trunk, which calculates to 36%. The superficial burn on her left arm is not included in the calculation. According to the Rule of Nines, the percentage in this patient can be calculated as follows: entire right arm = 9%; anterior right leg = 9%; anterior trunk 18%. 9+9+18= 36%. 9% (D), 18% (A) and 45% (C) are not accurate estimates of this patient's total body surface area of burns.
Which of the following patients should be started on isoniazid therapy? 21-year-old woman with a 5 mm PPD reaction and a history of IV drug abuse 40-year-old homeless man with a 10 mm PPD reaction 51-year-old doctor with a 5 mm PPD reaction 8-year-old boy with an 8 mm PPD reaction
40-year-old homeless man with a 10 mm PPD reaction Patients who are medically underserved (homeless, correctional institution residents) should be treated with isoniazid (INH) if their PPD reaction is >10 mm. The tuberculin skin test or PPD is the best tool available for detection of latent tuberculosis (TB). The antigens present in the PPD induce a reaction in patients with latent TB. After placement of the antigen, the patient has the reaction interpreted by a healthcare provider at 48-72 hours. Children <4 years of age should be treated if their reaction is >10 mm (D). Patients with a history of IVDA (A) and health care workers (C) should be treated if their PPD is >10 mm.
What is the Jarisch-Herxheimer reaction?
A febrile reaction to antigens that are liberated when spirochetal bacteria (classically syphillis) are destroyed by antibiotic therapy.
Which variant of acne vulgaris presents with fever and arthralgias with an acute eruption of large inflammatory nodules and friable plaques with hemorrhagic crusts?
Acne fulminans.
What is the other name for hidradenitis suppurativa?
Acne inversa.
A 16-year-old girl is in the clinic because of bumps on her face. She noted these for the past couple of weeks that seem to worsen whenever she gets her period. On physical examination, there are multiple open and closed comedones on the nose and multiple erythematous papules and pustules on the nose and cheeks. Which of the following is the most likely diagnosis? Acne rosacea Acne vulgaris Perioral dermatitis Sebaceous hyperplasia
Acne vulgaris The girl has examination findings consistent with acne vulgaris. Acne vulgaris is the most common cutaneous disorder affecting adolescents and young adults. It is a disease of pilosebaceous follicles. Four factors are involved: follicular hyperkeratinization, increased sebum production, Propionibacterium acnes within the follicle, and inflammation. The microcomedo is considered the precursor for the clinical lesions of acne vulgaris, including closed comedones, open comedones, and inflammatory papules, pustules, and nodules. Acne vulgaris typically affects those areas of the body that have the largest, hormonally responsive sebaceous glands, including the face, neck, chest, upper back, and upper arms. Young adolescents often have primarily comedonal acne consisting of noninflammatory lesions (closed or open comedones) involving the forehead, nose and chin. And as the acne progresses, patients develop inflammatory lesions (papules, pustules, and nodules). Common features of acne rosacea (A) include erythema, telangiectasias, and papules or pustules on the central face. There are no comedones in acne rosacea. Perioral dermatitis (C) is characterized by small, grouped, erythematous papules in a perioral (or occasionally perinasal or periorbital) distribution. When the perioral skin is involved, a rim of spared skin is usually seen around the vermilion border of the lip. Sebaceous hyperplasia (D) is due to visible enlargement of sebaceous glands. It most commonly occurs in adults with a history of oily skin. These growths are umbilicated yellowish papules are most commonly found on the forehead and cheeks.
A previously healthy 63-year-old woman presents to your office with a complaint of a painful rash that started two days ago. Physical exam shows a grouping of vesicles on an erythematous base just distal to the right scapula. There are no other skin lesions present. Which of the following is the most appropriate therapy? Acyclovir Amitriptyline Cephalexin Ganciclovir
Acyclovir Varicella-zoster virus (VZV) is the virus responsible for chickenpox and herpes zoster, also called shingles. Chicken pox is the initial infection that occurs after exposure to the virus. Herpes zoster occurs when the dormant virus is reactivated. Herpes zoster can occur at any age, but is more common in individuals older than 50 years. Clinical presentation of herpes zoster is a painful, unilateral, vesicular rash that occurs in a dermatomal distribution. In patients who are immunocompetent, the vesicles crust in 7-10 days and are no longer considered to be contagious at that point. Diagnosis is determined based on clinical presentation. Treatment decisions are based on duration of symptoms, clinical presentation, patient's age and immune state. Immunocompetent patients older than age 50 years who present within 72 hours of the onset of symptoms should be treated with an antiviral medication such as acyclovir (or valacyclovir) to help decrease symptoms and duration of the infection. The herpes zoster vaccine is recommended for individuals aged 60 years and older to help prevent herpes zoster infection. Amitriptyline (B) has been considered in the treatment of post-herpetic neuralgia. Due to increased side effects in the elderly, it is not recommended in the treatment of varicella zoster virus infections. Cephalexin (C) is used in the treatment of bacterial cellulitis. Ganciclovir (D) is an antiviral medication used to treat cytomegalovirus infections, not VZV infections.
What type of hypersensitivity reaction is allergic contact dermatitis?
Allergic contact dermatitis is a form of delayed hypersensitivity (type IV) reaction mediated by lymphocytes sensitized by the contact of the allergen with the skin.
A 27-year-old man presents with painful, swelling of his fingertip for three days as shown above. He also reports subjective fever and malaise. Which of the following represents the best course of management? Analgesia and a clean dressing Bedside incision and drainage Intravenous antibiotic administration and hospital admission Operative open irrigation and debridement
Analgesia and a clean dressing The patient has herpetic whitlow. This is a localized infection caused by the herpes simplex virus. Inoculation most commonly occurs when the patient touches herpetic lesions on the mouth or genitals. Physical examination reveals grouped vesicles on an erythematous base. Analgesia and a clean dressing minimizes the chance of transmission to other people or other areas of the body. Topical acyclovir may decrease the length of illness and oral acyclovir may decrease the risk of recurrence, but this is controversial.
A 24-year-old man presents to the ED with a rash on his left flank. He is an avid hiker in the upper Midwest. He was bit by a tick two weeks ago. What would you expect to find on physical examination? Annular erythematous patch with central clearing Diffuse erythroderma over the trunk and extremities Maculopapular rash over the trunk following Langer's lines Petechiae involving the palms and soles before spreading centrally
Annular erythematous patch with central clearing This patient is exhibiting risk factors for and signs of Lyme disease. Lyme disease is the most common vector-borne disease in the United States. It is endemic to New England, the mid-Atlantic states, and the upper Midwest. It is caused by the spirochete Borrelia burgdorferi and transmitted by the Ixodes dammini tick, more commonly known as the deer tick. The tick must be attached for more than 48 hours for transmission to occur. There are three stages of clinical Lyme disease. Early Lyme disease is characterized by erythema migrans, an annular erythematous patch with central clearing, which classically has a "bull's eye" appearance. Hematogenous spread leads to diffuse erythema migrans, which spares the palms and soles. Acute disseminated Lyme disease occurs approximately 4 weeks after initial infection and can include meningoencephalitis, Bell's palsy (which may be bilateral), or carditis which often manifests with AV block. Late Lyme disease develops greater than 1 year after initial infection and includes chronic arthritis with or without chronic subtle encephalopathy. Only 50% of patients remember a tick bite; thus, diagnosis may be difficult. Erythema migrans is diagnostic; however, not all patients present with this finding. Initial screening involves ELISA testing with Western Blot and PCR to confirm the diagnosis. If the diagnosis is suspected, empiric treatment should be administered. Treatment for early Lyme disease and mild acute disseminated Lyme disease is doxycycline. In pregnant women or children under the age of 8 years, amoxicillin should be substituted. Patients with neurologic or cardiac manifestations should be admitted and treated with IV ceftriaxone. Diffuse erythroderma over the trunk and extremities (B) is the characteristic rash of toxic shock syndrome. Petechiae involving the palms and soles before spreading centrally (D) is the characteristic rash of the tick-borne illness Rocky Mountain spotted fever. The petechial rash begins on the wrists and ankles, spreads to the palms and soles, and then spreads centrally to the trunk. A maculopapular rash over the trunk following Langers' lines (C) is the characteristic rash of pityriasis rosea. It is classically preceded by a herald patch and follows a dermatomal, "Christmas-tree pattern."
What medications may be used in the treatment of patients with hidradenitis suppurativa?
Antibiotics, retinoids, corticosteroids, anti-androgenic agents, estrogen derivatives, immunosuppresants and 5-alpha-reductase inhibitors have all been described for use in the treatment of hidradenitis suppurativa
Which of the following is concerning for melanoma? Areas of pigment regression Other similar lesions Pearly border Regular border
Areas of pigment regression Melanoma is a highly aggressive malignancy of melanocytes in the skin. Although it is the least common skin malignancy, it has the highest mortality. The peak incidence occurs in 20- to 45-year-olds which is much younger than the other two forms of skin cancer (basal cell carcinoma and squamous cell carcinoma). Any new mole after age 35 needs a dermatologic evaluation since people rarely form new moles at this age. The mnemonic ABCDE helps to remember the characteristics of melanoma: Asymmetry, Border (irregular), Color (different shades, not uniform), Diameter (>6 mm), Evolution. Areas of pigment regression describe changes in color and a lack of uniformity of the lesion. Other similar lesions (B) are atypical. Patients may have nevi or skin moles that appear similar, but as a lesion progresses to become melanoma it looks different from other moles on the body. A pearly border (C) is more consistent with a basal cell carcinoma. These lesions often begin as a pearly papule with telangiectasias. Regular borders (D) are uncommon as the malignancy does not grow in a uniform pattern.
A five-month-old boy presents to your ED with a rash. He has always had dry skin but the parents note that this rash seems worse. He has had no fevers but has been scratching his face. There is a strong family history of asthma and allergies. He is well appearing on exam with normal vital signs for his age. The blanching rash is located over his face, neck, and extensor surfaces. His diaper area is relatively spared. Which of the following is the most likely diagnosis? Asteatotic eczema Atopic dermatitis Contact dermatitis Dyshidrotic eczema
Atopic dermatitis Atopic dermatitis is a common chronic remitting disorder of dry skin that frequently frustrates parents and patients and is sometimes referred to as "the itch that rashes." It begins early in life between birth and six months of age and is characterized by papules and plaques that are erythematous and pruritic with occasional oozing, weeping, and crusting. It is often symmetrical in nature. There are three distinct phases that involve different locations despite a similar appearance of the lesions. In the infantile phase, lesions are found on the cheeks, forehead, trunk, and extensor surfaces. In the childhood phase (four to twelve years), lesions are found on the wrists, ankles, antecubital, and popliteal fossa. In the adult phase (> 12 years), lesions are mostly in flexural areas of the arms, neck, and legs, occasionally with lichenification. Treatment is frequent use of topical emollients and topical corticosteroids. Asteatotic eczema (A) is a form of dry skin that also appears as cracked skin with red fissures and scale and is usually seen in adolescents during the winter. Contact dermatitis (C) is caused by an inflammatory reaction to an allergen that has come in contact with the skin. It is usually not chronic in nature. Dyshidrotic eczema (D) is characterized by tiny clustered vesicles usually on the palms, soles, and lateral digits.
A 40-year-old man presents to your office with a complaint of a spider bite. He was chopping wood at his house, looked down, and saw a spider bite his leg. In addition to pain, his symptoms include muscle aches in his legs and back, severe abdominal pain, tremor, weakness, nausea, and vomiting. Which of the following is the most likely cause of his symptoms? Black widow spider Brown recluse spider Redback spider Wolf spider
Black widow spider Spider bites are rare and very few species of spider cause problems for humans. A number of more common disorders present with similar symptoms to a spider bite, such as skin infections and bites of other insects, therefore accurate diagnosis is the first goal in managing these patients. Black widow spiders are found throughout North America and can be identified with the classic red hourglass on the underside of the abdomen. These spiders are shy and solitary and generally are found in undisturbed places such as woodpiles, garages, or barns. They do not bite unless provoked. Widow spider venom is neurotoxic, and systemic symptoms include muscle aches, abdominal pain, tremor, weakness, nausea, and vomiting. Treatment approach depends on the severity of symptoms and response to initial supportive care. Antivenom is indicated for moderate to severe symptoms that do not respond to parenteral opioids and benzodiazepines. Patients with persistent severe pain and symptoms that do not respond to antivenom need hospitalization. Brown recluse spiders (B) are found in the geographic area encompassing central Texas to Tennessee and southern Illinois to Florida. Their venom is hemolytic and cytotoxic causing an intense inflammatory response. Within several hours of the bite, patients experience pain and pruritus with resulting ischemia and skin necrosis. The redback spider (C) is found in Australia and is a relative of the black widow spider found in North America. This spider is black with a red stripe on the dorsal aspect of the abdomen. Its venom is neurotoxic with bites causing localized pain, erythema, and some systemic symptoms including nausea, vomiting, headache, and arthralgias. Antivenom is available for severe cases. Wolf spiders (D) are found throughout the world, are fast, and can chase after prey. Wolf spiders generally run away when threatened and most are harmless. Bites cause pain that lasts a short time and very rarely cause systemic effects.
How are brown recluse spiders identified?
By the violin-shaped pattern on the cephalothorax.
Tinea
Capitis: children, kerion • Corporis: red, annular, raised, central clearing • Cruris: obese males, sweat • Pedis: sweaty feet • Unguium: raised, discolored nail; nail plate white, thick, crumbly • Capitis/unguium rx: PO antifungals
What syndrome can result from circumferential eschar from a full-thickness burn?
Compartment syndrome.
What is the causative organism of hand-foot-and-mouth disease?
Coxsackie virus.
A 23-year-old man presents with multiple skin papules on his inner thighs, groin, and penis. The papules are about 3 mm in diameter, have a central indentation, and are nontender. A small amount of caseous material can be expressed from the papules. Which of the following is the most appropriate therapy? Cryotherapy Oral acyclovir Reassurance and watchful waiting Topical mupirocin
Cryotherapy For patients who desire treatment or who have suspected sexually transmitted molluscum contagiosum, treatment options include cryotherapy, keratolytics, and curettage. Molluscum contagiosum is a local infection caused by a poxvirus. Molluscum contagiosum is commonly seen in children. Molluscum contagiosum can also be seen in adolescents and adults as a sexually transmitted disease or due to contact sports. Molluscum contagiosum is spread by direct skin-to-skin contact, through autoinoculation, or through contact with contaminated fomites. Molluscum contagiosum typically presents with firm, dome-shaped papules that are 2-5 mm in diameter. The papules are shiny and have a central umbilication. The lesions may occasionally be visibly inflamed. Molluscum contagiosum can occur anywhere on the body except for the palms and soles. Eyelid lesions can induce conjunctivitis. Sexually transmitted molluscum contagiosum usually appears on the groin, genitals, inner thighs, and lower abdomen. Laboratory studies are unnecessary in children, but sexually active adolescents and adults with genital lesions should be tested for other sexually transmitted diseases. Extensive lesions should raise the suspicion for human immunodeficiency virus infection. Options for treatment include benign neglect, cryotherapy, curettage, or keratolytics. Benign neglect is often recommended because molluscum contagiosum is typically benign and lesions heal within 1-2 months. Treatment is recommended for teenagers and adults with suspected sexually transmitted infections due to risk of transmission. Oral acyclovir (B) is not appropriate management in this patient. Acyclovir is an antiviral that is commonly used to treat genital herpes, herpes simplex infections, or herpes zoster. Antiviral medications such as cidofovir and ritonavir, are indicated for the treatment of molluscum contagiosum in immunocompromised patients. Reassurance and watchful waiting (C) is not appropriate management in teenage or adult patients with groin or genital lesions. These patients should be treated to decrease transmission to partners. Because treatment is often painful or tedious, reassurance and watchful waiting is appropriate management in children without distressing lesions. Topical mupirocin (D) is used for bacterial skin infections, such as impetigo. Antibiotics are ineffective against viral illness like molluscum contagiosum. Topical mupirocin would be appropriate if this patient develops signs of secondary bacterial infection of the lesions.
A patient presents to the emergency department with severe burns from a gas explosion. The burn appears waxy dry and red with blistering. It does not blanch with pressure and is only painful to pressure. Which of the following is the most accurate classification of burn in this patient? Deep partial-thickness burn Full-thickness burn Superficial burn Superficial partial-thickness burn
Deep partial-thickness burn This patient's presentation is most consistent with a deep partial-thickness burn. Current designations of burn depth are superficial, superficial partial-thickness, deep partial-thickness, and full-thickness. The term fourth degree is still used to describe the most severe burns, burns that extend into the muscle, bone, or joints. Superficial burns involve only the epidermal layer of skin. They do not blister but are painful, dry, red, and blanch with pressure. This process is commonly seen with sunburns. Partial thickness burns involve the epidermis and portions of the dermis. They are characterized as either superficial or deep. Superficial partial-thickness burns characteristically form blisters and are painful, red, and weeping, and blanch with pressure. Deep partial-thickness burns extend into the deeper dermis and damage hair follicles and glandular tissue. They are painful to pressure only, almost always blister, are wet or waxy dry, and have variable mottled colorization from patchy cheesy white to red. They do not blanch with pressure. These burns invariably cause hypertrophic scarring. Full-thickness burns extend through and destroy all layers of the dermis and often injure the underlying subcutaneous tissue. Burn eschar, the dead and denatured dermis, is usually intact. The eschar can compromise the viability of a limb or torso if circumferential. Full thickness burns are usually anesthetic or hypoesthetic. Skin appearance can vary from waxy white to leathery gray to charred and black. The skin is dry and inelastic and does not blanch with pressure. Vesicles and blisters do not develop. A full-thickness burn (B), superficial burn (C) and superficial partial-thickness burn (D) are not consistent with this clinical picture.
What is the prognostic indicator of a melanoma lesion?
Depth of invasion.
A 29-year-old woman presents to the ED with concerns regarding a "spot" on her arm that seems to have grown rapidly over the course of the last 2-3 months. The lesion is shown above. Which of the following is the most important prognostic factor in staging this lesion? Depth of the lesion Diameter of the lesion Lymph node involvement Pigment of the lesion
Depth of the lesion This patient has history and exam findings consistent with malignant melanoma, a highly aggressive malignancy of the pigment-producing cells (melanocytes) of the skin. This is the least common form of skin cancer; however, it holds the highest mortality. Risk factors include fair skin, sun exposure, and family history. It may arise de novo (70%) or from dysplastic nevi (30%). The most important prognostic factor for staging this malignancy is the depth of the lesion on biopsy. Signs and symptoms include new or changes to an existing mole and the ABCDE rule is used for diagnosis. The ABCDE rule consists of asymmetry of the lesion, border (irregular), color (not uniform), diameter (> 6 mm), and elevation or enlarging rapidly. The presence of these elements should raise your suspicion substantially for malignant melanoma. Formal diagnosis is made on skin biopsy. Management is surgical excision with wide margins. Early detection and excision is key and often curative. There is a much poorer prognosis with metastasis and lymph node involvement. Chemotherapy may be used for metastatic disease but has no effect on overall survival. While diameter of the lesion (B) and pigment of the lesion (D) are elements of diagnosis of malignant melanoma, neither is the most important prognostic factor in staging this lesion. Lymph node involvement (C) is an indicator of poor prognosis; however, it is not the most important overall prognostic factor in staging.
Which of the following is most suggestive of measles infection? A prodrome of fever, lymphadenopathy, and conjunctivitis followed by a maculopapular rash that starts on the face and spreads to the trunk and limbs Diffuse maculopapular rash with white spots on the buccal mucosa High fever for 3 days followed by the appearance of a pink maculopapular rash after defervescence Presence of shallow ulcers on oral mucosa and vesicular lesions on the palms and soles
Diffuse maculopapular rash with white spots on the buccal mucosa Measles (rubeola) is caused by a paramyxovirus and spread by respiratory contact. Measles begins with a prodrome of fever, coryza, cough, and conjunctivitis for several days followed by the development of a morbilliform rash. The rash starts with erythematous maculopapular lesions a few millimeters in diameter which become confluent. The rash typically starts on the face and spreads to the extremities. White spots on the buccal mucosa, known as Koplik spots, are pathognomonic for measles. Complications include otitis media, pneumonia, myocarditis, pericarditis, and encephalitis. Treatment is supportive. High fever for 3 days followed by the appearance of a pink maculopapular rash after defervescence (C) is classic for roseola. Roseola infantum, also known as exanthema subitum or sixth disease, is caused by human herpes viruses (HHV) 6 and 7. Affecting primarily infants and young children, it is characterized by high fever for 3 days which defervesces abruptly with the development of a pink maculopapular rash. A generally benign, self-limited condition, treatment is supportive. Presence of shallow ulcers on the oral mucosa and vesicular lesions on the palms and soles (D) is most suggestive of hand, foot and mouth disease. Commonly caused by coxsackie virus subtypes, hand, foot, and mouth disease is a highly contagious viral infection spread by fecal-oral route, respiratory droplets and contact with skin lesions. It begins with a prodrome of low-grade fever, anorexia, and mouth pain for 1-2 days prior to the appearance of the characteristic oral vesicular lesions which may ulcerate, and palm and sole lesions. A maculopapular rash may also be present. It is most common in preschool-aged children, and incidence peaks in summer and autumn. Treatment is supportive. A prodrome of fever, conjunctivitis, and lymphadenopathy followed by a maculopapular rash that starts on the face and spreads to the trunk and limbs (A) is suggestive of rubella infection. Rubella is caused by a togavirus and spread by respiratory droplets. Rubella begins with a prodrome of fever, lymphadenopathy, and conjunctivitis for 3 days followed by the development of a facial maculopapular rash that subsequent spreads to the trunk and limbs. Treatment is supportive and complications are unusual. However, primary infection with rubella in pregnancy can lead a constellation of severe symptoms in the developing fetus, and rubella is one of the "TORCH" infections. Transplacental rubella infection can lead to congenital rubella syndrome, which can result in sensorineural deafness, cataracts, cardiac malformations, and neurologic sequelae. Purpuric skin lesions associated with congenital rubella syndrome are known as "Blueberry muffin" spots.
A 25-year-old man presents to your office after having noticed a slightly raised red lesion with central clearing that increased in size over the past five days. He describes feeling flu-like symptoms and states that he had been camping in Connecticut recently. Based on the clinical presentation, which of the following is the best treatment option for this patient? Ceftriaxone 250 mg single dose intramuscular injection Doxycycline 100 mg oral tablet twice per day for 21 days Metronidazole 2 g single dose tablet orally Watch and wait
Doxycycline 100 mg oral tablet twice per day for 21 days Doxycycline 100 mg oral tablet twice per day for twenty-one days is the correct treatment and dosing for the suspected diagnosis of Lyme disease with a known erythema migrans (bulls-eye) rash. Lyme disease is caused by the spirochete Borrielia burgdorferi. It is transmitted to humans through a tick bite. The disease is most common in the northeastern United States, with Connecticut having one of the highest prevalence rates. It is recommended to treat any patient with suspected Lyme disease with an associated erythema migrans rash. Ceftriaxone 250 mg single dose intramuscular injection (A) is the correct treatment for urethritis caused by Neisseria gonorrhoeae. Gonococcal urethritis typically presents with painful urethral discharge. Patient's with urethritis should be treated concurrently for Chlamydial infection since the two infections commonly occur together. Metronidazole 2 g single dose orally (C) is the treatment for a trichamonal infection. Trichamonas is a flagellated protozoan, which infects the genitourinary tact of males and the lower urinary tract of females. Associated clinical findings include a malodorous, frothy, yellow-green discharge and a wet-mount that reveals motile flagellates. Watch and wait (D) is not the treatment of choice. Though it is suggested to send blood work for a confirmatory laboratory titer, the proper treatment course is to treat any patients with symptoms suggestive of Lyme disease. Delayed treatment can result in central nervous system and cardiac complications such as Bells' palsy, encephalitis, pericarditis, and heart block.
A 23-year-old woman presents to her primary care provider complaining of a recurring rash on her hands. The rash erupted suddenly and is extremely pruritic. She denies any new exposures. Physical exam reveals multiple, deep-seated vesicles scattered on both palms symmetrically. Which of the following is the most likely diagnosis? Bullous pemphigoid Contact dermatitis Dyshidrotic eczema Herpetic whitlow
Dyshidrotic eczema The woman in this case most likely has dyshidrotic eczema. Dyshidrotic eczema, also known as dyshidrosis or pompholyx, is dermatologic disorder characterized by intensely pruritic, vesicular eruptions on the palms or soles. Dyshidrotic eczema is most commonly seen in young adults. Recurrent episodes are common. Risk factors include a history of atopic dermatitis, exposures to allergens or irritants, intravenous immune globulin, smoking, and exposure to ultraviolet radiation. Dyshidrotic eczema episodes typically begin with pruritus, followed by acute eruption of extremely pruritic vesicles. The vesicles are usually deep-seated and often described as having a "tapioca pudding" appearance. In the majority of patients with dyshidrotic eczema, only the hands are involved. Vesicles last 2-3 weeks and then recur weeks to months later. Some patients report flares associated with emotional or physical stress. The diagnosis of dyshidrotic eczema is typically based on history and clinical findings alone; skin biopsy is rarely required. Treatment for mild to moderate disease involves high potency, topical corticosteroids that are applied two times per day for 2-4 weeks. Severe disease usually requires oral corticosteroids. Avoiding skin irritants and harsh soaps and applying emollients is an essential element of treatment. Recurrent episodes are less frequent with increasing age. Bullous pemphigoid (A) is a chronic, autoimmune disorder that is primarily seen in persons over the age of 60 years. Men are affected twice as frequently as women. Bullous pemphigoid presents with tense bullae that are most commonly located on flexural surfaces. Bullous pemphigoid is rarely localized on the hands or feet. Contact dermatitis (B) typically involves the dorsal aspect of the hand and the web spaces. There is often a history of direct irritant contact. Some of the most common causes of contact dermatitis include topical antimicrobials, anesthetics, hair-care products, nickel, and adhesive tapes. Patch testing may be useful for diagnosing allergic contact dermatitis. Herpetic whitlow (D) presents with painful, grouped vesicles on the lateral aspect of a finger. Herpetic whitlow is caused by the herpes simplex virus. Herpetic whitlow is usually unilateral, in contrast to dyshidrotic eczema, which is characteristically symmetrical and bilateral.
A 32-year-old man presents to the office with a rash on both of his hands. The rash appeared a couple days after completing yard work. He wore gloves while working and reports no similar rashes in household contacts. There are pink patches with multiple tapioca-like vesicles involving primarily the palms and web spaces on both hands. Which of the following is the most likely diagnosis? Contact dermatitis Dyshidrotic eczematous dermatitis Rhus dermatitis Scabies
Dyshidrotic eczematous dermatitis Dyshidrotic eczema is an intensely pruritic, chronic recurrent dermatitis, typically involving the palms and soles. It starts as an episode of intense itching, followed by the formation of small vesicles. The vesicles are described as tapioca-like on the lateral aspect of the fingers and show confluence. Desquamation occurs over 1-2 weeks, leaving fissures and erosions. Treatment includes a high-potency topical steroid and prevention of secondary infection. Contact dermatitis (A) would be a reasonable choice if the patient had not been wearing gloves. Contact dermatitis is most often diagnosed based on the distribution of the rash and a history of exposure to irritants or allergens. Rhus dermatitis (C), also known as toxicodendron dermatitis, is a specific type of contact dermatitis to oil produced by plants in the genus Toxicodendron (poison ivy, poison oak and poison sumac) among others. Scabies (D) is a parasitic skin infection caused by the mite Sarcoptes scabiei. Scabies is a pruritic rash that can affect areas of the body with skin folds, such as the web spaces of the hands. Superficial burrows are created as the adult mite excavates through the epidermis. These burrows are accompanied by mosquito-like "bites."
The superficial fungal infections are most commonly caused by dermatophytes in what genera?
Epidermophyton, Trichophyton, and Microsporum genera.
A 65-year-old woman who is morbidly obese presents to your office with intertrigo in the axilla. On examination you detect small, reddish-brown macules that are coalescing into larger patches with sharp borders. You suspect a secondary infection complicating the intertrigo. What is the most appropriate topical treatment for this condition? Erythromycin Mupirocin Nyastatin Zinc oxide
Erythromycin Intertrigo is a superficial inflammatory dermatitis occurring on two closely opposed skin surfaces as a result of moisture, friction, and lack of ventilation. Bodily secretions, including perspiration, urine, and feces, often exacerbate skin inflammation. Physical examination of skin folds reveals regions of erythema with peripheral scaling. Excessive friction and inflammation can cause skin breakdown and create an entry point for secondary fungal and bacterial infections, such as Candida, Group A beta-hemolytic streptococcus, and Corynebacterium minutissimum. Cutaneous erythrasma is caused by Corynebacterium minutissimum and presents as small reddish-brown macules that may coalesce into larger patches with sharp borders. They may be asymptomatic or pruritic and fluoresces coral-red on Wood lamp examination. Intertrigo complicated by erythrasma is treated with topical or oral erythromycin. In uncomplicated intertrigo, numerous agents and mechanisms can be used to keep the skin folds dry, clean, and cool. Applying barrier protectants reduces skin breakdown and alleviates pruritus and pain. Zinc oxide (D) can be used as a skin protectant to prevent further skin breakdown and introduction of secondary infections. Separating skin surfaces with absorbent products, such as gauze, cotton, and products with water vapor-permeable sheets, may also help reduce friction. Intertrigo complicated by fungal infection should be managed with topical antifungals. Nystatin (C) is effective only for candidal intertrigo. The diagnosis of secondary fungal infections is commonly made clinically, based on the characteristic appearance and distribution of satellite papules and pustules. The diagnosis may, however, be confirmed with a potassium hydroxide preparation positive for pseudohyphae and spores. Additionally, a potassium hydroxide preparation, Wood lamp examination, or culture of skin scrapings can diagnose conditions such as Candida or dermatophyte infections. The optimal treatment for patients with intertrigo complicated by group A beta-hemolytic streptococcal infections includes single or multiple regimens of topical therapies such as mupirocin (B).
A 27-year-old woman presents complaining of recurrent skin infections along her axilla since age 19. Prior incision and drainage coupled with short courses of antibiotics have resulted in temporary resolution of the skin lesions, only to have them recur again. Physical examination reveals an obese female with two 1 cm erythematous papules along the right axilla. The left axilla is notable for cord-like scarring and double blackheads, without any actively erythematous lesions. A culture obtained from one of the axillary lesions shows no bacterial growth. Which of the following is true about this condition? Effective management consists primarily of repeated incision and drainage Evidence supports an association between smoking and severity of disease Hormonal factors appear unrelated to disease severity and remission This condition is more common in men than women
Evidence supports an association between smoking and severity of disease This patient has hidradenitis suppurativa (HS), a chronic skin condition involving recurrent painful, erythematous nodules, occurring most commonly in the axilla and groin that results from occlusion of follicles and secondary inflammation of the apocrine glands. Lesions may also occur along the perineum, perianal region, abdomen and breasts. The lesions may exhibit purulence, and can lead to chronic scarring and the development of sinus tracts. The cause of HS is unclear; theories include an abnormal immune response of the skin may play a role. Smoking has been found to worsen disease severity and frequency of recurrence. Treatment is challenging, with topical antibiotics, antiandrogens, and oral antibiotics being the mainstay of therapy. Isotretinoin and surgical excision are all used, typically with limited results. Hormonal factors (C) do appear to be related to disease severity and remission; some women experience pre-menstrual flares and find that remission occurs after menopause. This condition is more common in women than in men (D). Repeated incision and drainage (A) is usually inadequate for lesion resolution. Effective management requires a comprehensive approach including lifestyle changes such as smoking cessation and weight loss, medical therapy, and surgical treatment in cases of more severe disease.
True or false: surgical removal of a nail infected with onychomycosis provides a definitive cure?
False.
Amitriptyline (B) has been considered in the treatment of post-herpetic neuralgia. Due to increased side effects in the elderly, it is not recommended in the treatment of varicella zoster virus infections. Cephalexin (C) is used in the treatment of bacterial cellulitis. Ganciclovir (D) is an antiviral medication used to treat cytomegalovirus infections, not VZV infections.
Famciclovir and valacyclovir.
A 19-year-old woman presents to her primary care provider's office for a rash on the back of her neck that has grown and gotten darker over the past year. The woman has a past medical history asthma. Her pulse is 66 beats/min, respiratory rate is 18 breaths/min, blood pressure is 132/87 mm Hg, and her body mass index is 29. On exam, there is a horizontal, dark, velvety plaque on her posterior neck. Which of the following diagnostic studies is most appropriate? Fasting blood glucose level Punch biopsy of the plaque Serum antinuclear antibody level Serum cholesterol level
Fasting blood glucose level A fasting blood glucose level should be obtained to evaluate for diabetes mellitus, which is frequently associated with acanthosis nigricans. Acanthosis nigricans is a common skin condition that is characterized by velvety, hyperpigmented plaques. Acanthosis nigricans recognition is important because of its association with several systemic diseases. Obesity, insulin resistance, and diabetes mellitus are the most commonly associated disorders. Internal malignancy may rarely cause acanthosis nigricans. Acanthosis nigricans is more common in Native Americans, African Americans, and Hispanics than in Caucasians or Asians. Hyperinsulinemia is thought to play a major role in the development of the hyperpigmented plaques by stimulating keratinocyte proliferation. Acanthosis nigricans typically presents as a thick, brown, horizontal plaque on the back of the neck or axillae. Early plaques have a dirty appearance that may become thicker and darker as the disease progresses. Acanthosis nigricans typically has a symmetrical distribution. Patients with acanthosis nigricans should under go evaluation for associated diseases like type 2 diabetes mellitus, polycystic ovarian syndrome, and metabolic syndrome. Diagnostic studies may include fasting blood glucose, hemoglobin A1C, and postprandial blood glucose. In older, non-obese patients, acanthosis nigricans should raise suspicion for underlying malignancy. In some cases, the development of acanthosis nigricans precedes recognition of the malignancy by several years. Features that suggest underlying malignancy include rapid onset of plaques, extensive involvement, atypical sites, unexplained weight loss, and older adults. Treatment of the underlying cause frequently causes regression and fading of acanthosis nigricans. For patients who have unsatisfactory improvement or who need accelerated improvement, topical retinoids and topical vitamin D analogs may be beneficial. Because acanthosis nigricans is a benign skin disorder and can be clinically diagnosed, punch of biopsy of the lesion (B) is unnecessary. A punch biopsy may be used in the evaluation of skin disorders like psoriasis, lichen planus, lichen sclerosus, or malignancies. Serum antinuclear antibody level (C) is typically used in the evaluation of autoimmune and rheumatologic conditions. Antinuclear antibodies bind to autoantigens present in the nuclei of mammalian cells. Antinuclear antibodies are found in diseases such as systemic lupus erythematosus, rheumatoid arthritis, scleroderma, and polymyositis. Serum cholesterol level (D) is not indicated at this time. However, because obesity and diabetes mellitus are risk factors for hypercholesterolemia, serum cholesterol should be monitored in this patient.
What is the most common complaint of patients with lupus?
Fatigue.
An 18-year-old man is diagnosed with condyloma acuminatum in the genital area. Which of the following physical exam findings is most consistent with this diagnosis? Flesh-colored, exophytic lesions Painless ulceration Shallow, painful ulcerations Umbilicated, dome-shaped lesions
Flesh-colored, exophytic lesions Condyloma acuminata, commonly known as genital warts, can appear as flesh-colored, exophytic lesions or can appear as small bumps that are flat, pedunculated, verrucous, or papilliform. Condyloma acuminatum is caused by an infection with the human papillomavirus (HPV) types 6 and 11. It is a sexually transmitted infection that can be prevented with vaccination. All girls and boys who are 11 or 12 years old should get the recommended series of HPV vaccine regardless of sexual history. HPV is strongly associated with the development of genitourinary and rectal cancer. The infection can affect the vagina, cervix, vulva, oropharynx, perineum, and perianal areas. Treatment includes trichloroacetic acid solution, podophyllin, cryosurgery, surgical excision, or imiquimod cream application. Painless ulcerations (B) are associated with syphilis. Shallow, painful ulcerations (C) are associated with genital herpes. Raised lesions with an umbilicated center (D) are associated with molluscum contagiosum.
Which of the following is true regarding the use of tissue adhesives for wound repair? Frequent prolonged exposure to water should be avoided to prevent premature breakdown of the tissue adhesive Standard wound care, including use of topical antibiotic ointment, should be performed on lacerations closed with tissue adhesives Tissue adhesives can be used to close lacerations on mucosal surfaces Tissue adhesives can be used to close lacerations that occur as a result of animal bites
Frequent prolonged exposure to water should be avoided to prevent premature breakdown of the tissue adhesive Tissue adhesives can break down prematurely with frequent prolonged exposure to water. Therefore patients should be instructed to avoid soaking the affected area and activities such as swimming. Patients can shower and pat the area dry, however scrubbing and vigorous washing should be avoided. Tissue adhesives, or skin glues, are cyanoacrylates formulated for use on human skin. They are indicated for closure of linear, non-contaminated lacerations to the face, extremities and torso. Generally they are effective on wounds with little tension that would be amenable to repair with 5-0 sized suture material. Benefits of tissue adhesives include: ease and speed of application, minimal to no patient discomfort with application, reduction in needle-stick risk to healthcare providers, provides a protective covering to the wound and eliminate need for follow-up for suture removal. Cosmetic result is similar if not better than repair with sutures. Wound strength is initially less with skin glue closure, however after 7 days is comparable to sutured wounds. Tissue adhesive breaks down with in 5-10 days and the material will slough off without intervention. Contraindications to the use of skin glue include: jagged or irregular-shaped lacerations; wounds sustained as a result of an animal or human bite (D), puncture or crush injury; contaminated or infected wounds; wounds on mucosal surfaces (C); wounds in high moisture areas (axilla, perineum, palms); wounds overlying joints (unless they are immobilized). Tissue adhesives are broken down by the application of ointments (B), such as triple antibiotic or bacitracin. Therefore patients should be instructed to avoid putting these on lacerations that are repaired with skin glue. If removal is needed, antibiotic ointment and petroleum jelly are effective in aiding in removal.
Which type of malignancy is most associated with malignant acanthosis nigricans?
Gastric cancer.
Which gastrointestinal malignancy is associated with acanthosis nigricans?
Gastric carcinoma.
Which two HPV strains are commonly associated with cervical cancer and are therefore targeted in HPV vaccines?
HPV types 16 and 18.
What histological finding is typical of molluscum contagiosum lesions?
Henderson-Paterson bodies.
A 66-year-old woman presents with a rash to the right side of her forehead that began 2 days ago. She describes it as painful, blistering, and weeping. On exam, you note the rash seen above. Which of the following is the most likely diagnosis Acne rosacea Contact dermatitis Herpes zoster Impetigo
Herpes zoster Zoster ophthalmicus is due to reactivation of latent varicella virus in the dorsal root ganglia. It is characterized by a painful rash described as papulovesicular eruptions on an erythematous base along a dermatome. The lesions are often preceded by pain in a dermatomal distribution 2-3 days before the characteristic rash appears. Lesions often involve the face, mouth, eyes, ears, or tongue when branches of the trigeminal nerve are affected. Involvement of the tip of the nose is referred to as Hutchinson's sign and often signals concurrent ocular involvement of the nasociliary branch. Ramsey-Hunt Syndrome refers to involvement of the external auditory canal and development of an ipsilateral facial palsy. Treatment of zoster ophthalmicus involves analgesics, anti-inflammatories, and anti-viral medications (acyclovir or valacyclovir). The most common complication secondary to herpes zoster is post-herpetic neuralgia (chronic pain) of in the area of the involved dermatome. Acne rosacea (A) is an inflammatory disorder characterized by erythema, telangiectasias, and pustules primarily affecting the central face. It is primarily seen in women over the age of 30, but can be seen in men also. Contact dermatitis (B) is a rash caused by an exposure to either an irritant or allergen. The rash can be either chronic or acute and can have many different appearances depending on the type and length of exposure. Impetigo (D) is a superficial infection of the skin caused by S. aureus or beta-hemolytic streptococci. It is characterized by honey-colored, crusted lesions
An 18-year-old obese woman presents to your office with a complaint of redness and pain in her right axilla. Physical exam reveals a solitary nodule, approximately 2 cm in size, with surrounding inflammation and erythema. She tells you that she's had similar "boils" in the past. Which of the following is the most likely diagnosis? Contact dermatitis Dermoid cyst Granuloma inguinale Hidradenitis suppurativa
Hidradenitis suppurativa Hidradenitis suppurativa (HS) is a chronic inflammatory skin condition that causes scarring, keloids, contractures and immobility. Originally believed to be caused by a defect in the apocrine glands, it may also be due to a defect in the follicular epithelium. HS begins in adolescence or adulthood in otherwise healthy individuals. Risk factors for the development of HS include obesity, genetics, smoking, diet and mechanical stress on the skin. Onset is insidious, with the first presentation generally being erythema in an intertriginous skin area, most commonly the axilla. As the disease progresses, formation of sinus tracts, multiple open comedomes and scarring occur. The initial presentation can mimic other disorders and individuals often are diagnosed incorrectly with recurrent furunculosis. Diagnosis is a clinical one. Treatment includes medical management in the early stages and surgical intervention after the formation of abscesses or sinus tracts. Contact dermatitis (A) presents in a varying manner depending on severity, from mild skin dryness and erythema to skin necrosis. It is caused by an allergic reaction to a physical or chemical irritant. Dermoid cyst (B) affecting the skin is a solitary subcutaneous cyst that is congenital and may contain fatty masses or hair. Granuloma inguinale (C) is a chronic bacterial infection associated with sexually transmitted diseases. Common presentation is of a painless, beefy red ulcer in an intertriginous skin area.
A sexually active 17-year-old woman presents to her primary care clinic complaining of the lesions seen above around her genital area. Which of the following is the cause of these genital lesions? Herpes simplex virus Human papillomavirus Poxvirus Treponema pallidum
Human papillomavirus Condyloma acuminatum are the most common form of viral genital mucosal lesions and are caused by several types of human papillomavirus (HPV). The majority of genital warts are caused by HPV strands 6 and 11. The infection manifests as painless, verrucous fleshy papules. The warts may be located anywhere in the anogenital area. Lesions usually appear within weeks to months after exposure to HPV. They are generally asymptomatic, but may be painful, friable, or pruritic. Syphilis is caused by the spirochete Treponema pallidum (D) and is sexually transmitted. The genital lesion of primary syphilis is known as a chancre and is characteristically painless. These lesions appear as ulcerations, not wart-like. Herpes simplex virus (A) is responsible for genital and oral herpes. It presents as recurrent small group vesicles on an erythematous base, especially in the orolabial and genital areas. Patients commonly complain of burning and stinging sensation as well as neuralgia that may precede or accompany attacks. Molluscum contagiosum, caused by a poxvirus (C), presents as singular or multiple dome-shaped, waxy papules 2-5 mm in diameter that are umbilicated. The principle sites affected are the face, lower abdomen, and genitals.
What is the lethal disease that results from the parvovirus in pregnancy?
Hydrops fetalis.
A 17-year-old girl with no past medical history presents with pain and swelling to the left axilla for 3 days. She states this has never happened before. Physical examination reveals a 3 cm area of swelling and tenderness in the axilla with no warmth or erythema. The area is fluctuant. Which of the following represents the appropriate management? Amoxicillin and follow up Ciprofloxacin and follow up CT scan with contrast Incision and drainage
Incision and drainage The patient presents with an uncomplicated, simple cutaneous abscess, which should be treated with incision and drainage (I&D). A simple cutaneous abscess is a localized collection of pus that results in a fluctuant soft tissue mass. They can occur in any part of the body but are most commonly found in the neck, axilla and perirectal areas. Most abscesses contain bacteria but up to 5% of them are sterile. This is most commonly seen in patients with parenteral drug abuse. The majority of cutaneous abscesses are caused by skin flora and Staphylococcus aureus is the most commonly cultured aerobic agent. Abscesses near mucous membranes (perioral and perirectal) tend to predominantly be caused by anaerobic bacteria. The standard treatment for cutaneous abscesses is incision and drainage. In patients with normal immune systems and minimal erythema after drainage, antibiotics are not indicated. For I&D, the incision should be made to adequate length and depth to ensure complete drainage. The cavity is typically left open with or without a wick for drainage. Treatment with antibiotics alone (A & B) is inadequate as the antibiotic will not be able to adequately penetrate the abscess cavity to eliminate the infection. CT scan with contrast (C) is not indicated in the workup of a simple cutaneous abscess.
A 22-year-old man presents to the emergency department after being bitten by a shiny, black spider. He complains of severe abdominal pain and cramping. On exam, there is intermittent muscle rigidity involving his right arm, abdomen, and back. There is a pale circular patch around a central punctum and surrounding erythema on the posterior surface of his right hand. Which of the following is the most appropriate treatment? Black widow antivenin Intravenous morphine and diazepam Intravenous sodium bicarbonate Oral cephalexin and wound debridement
Intravenous morphine and diazepam Intravenous morphine and diazepam is the most appropriate treatment for this man with moderate widow spider envenomation. In the United States, widow spiders (genus Latrodectus) are one of the most commonly implicated species in envenomations. Black widow spiders are commonly encountered outdoors in cluttered areas, such as garages, wood piles, or sheds. People are commonly bitten while putting on shoes or gloves, gathering or chopping wood, or moving garbage. The majority of bites are on the extremities. The black widow venom is an excitatory neurotoxin that triggers large exocytosis from presynaptic nerve terminals. Patients typically present acutely with pain around the bite and severe abdominal pain. Vital signs are usually within normal limits. Intermittent muscle rigidity adjacent to the bite or involving the abdomen, chest, or back is present in more than half of patients. The bite appears as a circular area of pallor with a surrounding erythematous perimeter and a central puncture. Laboratory studies are unnecessary and are non-specific. Additional diagnostic imaging may be required to rule out abdominal surgical emergencies that can also cause severe abdominal pain and rigidity. Patients with mild envenomation can be managed with local wound care, oral analgesics, and tetanus prophylaxis. Moderate to severe envenomation treatment involves wound care, tetanus prophylaxis, parenteral opioids, and parenteral benzodiazepines. Intravenous calcium is not generally recommended. Widow antivenom is recommended for patients with severe envenomation that are unresponsive to parenteral analgesic and benzodiazepine therapy. Most patients with widow spider bites can be managed on an outpatient basis. Pain typically resolves within 24-72 hours if untreated. Black widow antivenin (A) is not yet indicated in this patient. Antivenin is reserved for patients unresponsive to opioids and benzodiazepines. Indications for antivenom include severe and persistent pain and muscle cramping, hypertension and tachycardia, or difficulty breathing. Black widow antivenom has the potential to cause anaphylaxis and is a pregnancy class C. Asthma and allergy to horses are relative contraindications. Intravenous sodium bicarbonate (C) are not used in the management of black widow envenomation. Sodium bicarbonate is used in the treatment of tricyclic antidepressant and salicylate overdoses and metabolic acidosis. Oral cephalexin and wound debridement (D) may be required later in the clinical course if an infection develops. Empiric antibiotic use is not recommended. If cellulitis develops, antibiotics should be directed against skin pathogens.
What is the medical term for the manifestations of widow spider bites?
Latrodectism
Which of the following statements is true regarding the condition seen above? Leads to scarring alopecia Primarily due to bacterial infection Requires long-term topical treatment with antifungal Usually painless
Leads to scarring alopecia The image depicts a patient with a kerion. This condition begins as Tinea capitis (scalp ringworm) that undergoes a delayed-type hypersensitivity reaction to the causative fungus. This inflammation causes the initial erythematous, scaly plaque of Tinea capitis to become boggy with inflamed purulent nodules and plaques. The hair follicle is frequently destroyed by the inflammatory process in a kerion, leading to scarring alopecia. Treatment includes long-term systemic therapy, usually with oral griseofulvin and the addition of an antibiotic to treat any secondary bacterial infection. In addition, oral corticosteroids are administered to treat the severe inflammation. Tinea capitis is a dermatophyte (B) infection most commonly caused by Trichophyton tonsurans transmitted from person to person via fomites, such as a barber's razor. Treatment with topical agents (C) is inadequate; patients require systemic therapy. The initial infection of Tinea capitis is usually painless (D) with intense pruritus at times. However, with progressive inflammation and the development of a kerion, the lesion becomes painful.
Melanoma
Least common skin malignancy but highest mortality • ABCDE: Asymmetry, Border (irregular), Color (different shades, not uniform), Diameter (>6 mm), Evolution
What are the unique laboratory findings of pleural effusions associated with rheumatoid arthritis?
Low glucose, low pH and high LDH.
An 18-year-old woman presents complaining of perioral dermatitis that she has experienced for the past two weeks. She states it has recently started itching and burning. What is the best treatment for this condition? Isotretinoin cream Metronidazole gel Oral prednisone Triamcinolone cream
Metronidazole gel Perioral dermatitis is defined by discrete erythematous micro-papules and microvesicles that concentrate in the perioral, and occasionally periorbital, area. These lesions often last weeks to months and sometimes cause an itching or burning sensation. It may occur in all ages, but affects women more commonly than men. Diagnosis is clinical, though it might be beneficial to culture the lesions to rule out a S. aureus infection. While the etiology is unknown, perioral dermatitis is known to be aggravated by topical glucocorticoids. Metronidazole gel or erythromycin gel is first-line topical treatment for perioral dermatitis. Isotretinoin cream (A) is used to treat severe acne, not perioral dermatitis. It works by inhibiting sebaceous gland function, and is a known teratogen. Oral prednisone (C) works to systematically reduce inflammation, but is not indicated to treat perioral dermatitis. Triamcinolone cream (D) is a topical steroid that has been shown to exacerbate perioral dermatitis.
Are skin glues toxic to the eye?
No.
In what circumstance is dry heat recommended in the treatment of frostbite?
None. Dry heat may cause further injury.
What are 3 predisposing factors for intertrigo?
Obesity, diabetes mellitus and HIV.
A 72-year-old woman complains for discolored toenails that have been present for years and has spread toward the cuticles from the distal corners. She tries to wear well fitting shoes and keeps her nails trimmed. On physical exam, the nail of the great toe is yellow-brown in color and appears thickened. The proximal end of the nail appears normal. Which of the following is the most likely diagnosis? Onychogryphosis Onychomycosis Subungual hematoma Tinea pedis
Onychomycosis This woman most likely has onychomycosis. Onychomycosis is fungal infection of the nail. Onychomycosis can be caused by fungi, yeasts, and nondermatophyte molds. Fingernail onychomycosis is most commonly caused by yeast. Onychomycosis is more common in adults than in children and is more common in men than in females. There are several subtypes of onychomycosis, but the most common clinical subtype is distal lateral subungual onychomycosis. Over 80% of onychomycosis cases in the United States are caused by Trichophyton rubrum. Risk factors for onychomycosis include increased age, swimming, tinea pedis, psoriasis, diabetes, immunodeficiency, and living with individuals with onychomycosis. Distal lateral subungual onychomycosis often begins on the great toe spreading from a distal corner to the cuticle. The nail appears yellowish or brownish and is often thickened and rough. Although onychomycosis is primarily a cosmetic concern for most patients, occasionally severe disease can cause discomfort or pain. The diagnosis of onychomycosis can be confirmed with either a KOH preparation or periodic-acid-Schiff staining. Treatment of onychomycosis is recommended in the following populations: patients with a history of lower extremity cellulitis, patients with diabetes and cellulitis risk factors, immunosuppressed patients, and patients with cosmetic concerns. Treatment primarily consists of topical and systemic antifungal drugs. Systemic therapy has a higher cure rate and shorter treatment courses but may be inappropriate in certain patients. Onychogryphosis (A) is the development of thickened and distorted nails with increased curvature. Onychogryphosis is more commonly seen in the elderly and in patients with poor nail care or ill-fitting footwear. A subungual hematoma (C) is a collection of blood underneath the nail bed due to an acute trauma. Subungual hematomas appear as a dark brown or maroon area underneath the nail. Tinea pedis (D) refers to a dermatophyte infection of the foot dermis. Tinea pedis is characterized by erythema, maceration, fissuring, and scaling that is often observed between the digits. Tinea pedis is often associated with onychomycosis.
One of the mainstay treatments for patients diagnosed with systemic lupus erythematosus is an antimalarial drug hydroxychloroquine. Prior to starting a patient on this medication, which of the following baseline screening tests or exams is recommended? Assessment of bone mineral density Mammogram Ophthalmologic exam Papanicolaou test (Pap test)
Ophthalmologic exam Early diagnosis and treatment of systemic lupus erythematosus (SLE) is important in preventing tissue and organ damage, achieving remission, and ensuring long-term survival. An antimalarial drug, hydroxychloroquine, is the mainstay of treatment for patients with lupus. Hydroxychloroquine reduces lupus symptoms, reduces mortality, decreases incidence of diabetes, and has antithrombotic and favorable lipid effects. Hydroxychloroquine has high tolerability compared with corticosteroids.The long-term use of hydroxychloroquine may lead to retinal toxicity, which is why a baseline ophthalmologic examination is recommended when patients start hydroxychloroquine and every 6 - 12 months while taking this medication. Mammogram (B) and Papanicolaou test (D) are not required before starting patients on hydroxychloroquine, however, they are recommended for patients with SLE due to increased risk of cancer as a result of immunosuppressive therapy. Assessment of bone mineral density (A) is recommended for patients with lupus, especially those taking high-dose steroids for prolonged period of time as they are at increased risk for osteoporosis.
A previously healthy 65-year-old woman presents to your office with a complaint of thickened and discolored toenails. Previous potassium hydroxide examination of her toenail scrapings shows the presence of dermatophytes. Which of the following is the most appropriate therapy? Oral fluconazole Oral terbinafine Topical medicated chest rub Topical terbinafine
Oral terbinafine Onychomycosis is a fungal infection that can affect the toenails or fingernails. There are five different subtypes of onychomycosis that are determined based on clinical features. Patients may present with a combination of these subtypes. Risk factors for onychomycosis include older age, diabetes mellitus, tinea pedis, genetic predisposition, and living with individuals who have onychomycosis. Patients initially present with complaints about the appearance of the nail without physical symptoms. With progression of the infection, thickening, discoloration and deformation of the nails occur and can lead to physical discomfort and pain. Diagnosis is with potassium hydroxide (KOH) examination of nail scrapings. Treatment is indicated for patients with diabetes, with a history of cellulitis, those experiencing pain or physical discomfort and when requested due to cosmetic concerns. First-line treatment is with oral medication. Patients should be advised that recurrence is common. Terbinafine has been found to have the highest cure rate and is taken for six weeks when treating fingernails, twelve weeks when treating toenails. Serum aminotransferases should be monitored before starting treatment with terbinafine and during the treatment process due to hepatotoxicity. Oral fluconazole (A) is sometimes used in the treatment of onychomycosis, but is not as effective as terbinafine and is therefore not recommended as first-line treatment. Topical agents are sometimes used as an adjunct to oral treatment. Medicated chest rubs (C) have been used anecdotally and are unlikely to be harmful, however additional studies need to occur before their use can be recommended. Topical terbinafine (D) is not recommended as first-line therapy for onychomycosis.
What is the recommended systemic treatment for perioral and periorbital dermatitis?
Oral tetracyclines, such as doxycycline.
A 30-year-old daycare worker has a 3-day history of arthralgias, malaise, and a rash. The rash is nonpruritic, maculopapular, irregular, and is located on her thighs and inner aspects of her upper arms. Symmetric synovitis is present in her fingers. Small effusions, warmth, and tenderness are noted in her left wrist and right elbow. No other joints are affected. Which of the following is most likely responsible for this presentation? Adenovirus Measles (rubeola) virus Parvovirus B19 Varicella-zoster virus
Parvovirus B19 Also known as erythema infectiosum or Fifth disease, parvovirus B19 infection usually is asymptomatic or causes mild, nonspecific, cold-like symptoms. However, several clinical conditions have been linked to the virus. Parvovirus B19 usually infects children and causes the classic "slapped-cheek" rash of erythema infectiosum (fifth disease). The virus may also cause acute or persistent arthralgias and papular, purpuric eruptions on the hands and feet in adults. This infection should be particularly suspected in health-care workers who have frequent contact with children. Parvovirus B19 infection can also trigger an acute cessation of red blood cell production, causing transient aplastic crisis, chronic red cell aplasia, hydrops fetalis, or congenital anemia. Adenovirus (A) is a mild virus that causes fever, conjunctivitis, pharyngitis, rash and can manifest as gastroenteritis. It is prevalent in both the pediatric and adult population and treatment is symptomatic. Measles (rubeola) virus (B) is characterized by cough, coryza, conjunctivitis, maculopapular rash and the pathognomonic oral enanthem known as Koplik spots. Measles virus and adenovirus do not cause arthritis. Varicella-zoster virus (D) also known as chickenpox typically presents with fever, malaise and a widespread vesicular and pruritic rash located on the torso and face. This virus was once common in children before the advent of the varicella vaccine in the United States. Adults, pregnant women, immunocompromised patients and neonates are at high risk of complications from infection with varicella such as pneumonia, hepatitis and neurologic sequelae. Varicella-zoster virus may cause large joint arthritis, but the rash is distinctively vesicular and pruritic.
Molluscum Contagiosum
Patient will be a school-age child • Complaining of "warts" • PE will show multiple waxy, dome-shaped papules with umbilicated appearance • Most commonly caused by Poxvirus • Treatment is self-limiting
Hidradenitis Suppurativa
Patient will be a women • With a history of lesions that have waxed and waned over the past few years • Complaining of tender nodules in her axillae and anogenital area • PE will show lesions that are tender, malodorous, often with exudative drainage, sinus tracts • Treatment is intralesional triamcinolone, topical clindamycin • Comments: Hurley staging system describes the severity of disease
Acne Vulgaris
Patient will be an adolescent • Complaining of rash on face, neck, upper chest and back • PE will show closed comedones (whiteheads), open comedones (blackheads), papules and pustules • Most commonly caused by Propionibacterium acnes • Treatment is: . Mild to moderate: topical retinoids, topical antibiotics, or benzoyl peroxide . Moderate to severe: add oral antibiotics . Severe: oral isotretinoin (Pregnancy class X, must have two forms of birth control)
Measles (Rubeola)
Patient will be an unvaccinated young child • With a history of a maculopapular rash that started on head and spreads towards feet • Complaining of high fever, cough, conjunctivitis, coryza • PE will show red spots with blue/white center on buccal mucosa (Koplik spots) • Diagnosis is made clinically • Treatment is supportive care
Dyshidrotic Eczema
Patient will be complaining of intense pruritus on their palms and sides of the fingers • PE will show vesicles that appear to contain "grains of tapioca" • Treatment is avoidance of long exposure of the hands to water, topical corticosteroids for acute flares
Vitiligo
Patient will be complaining of patches of pale skin affecting the neck, upper back and the chest • PE will show white, non-scaling, well-demarcated areas of hypopigmentation • Most commonly caused by autoimmune destruction of melanocytes • Comments: Wood's lamp examination will accentuate the hypopigmentation
Scabies
Patient will be complaining of severe pruritus that is worse at night • PE will show small papules, vesicles and burrows in the webbed spaces of the fingers and toes • Diagnosis is made by microscopic visualization • Most commonly caused by Sarcoptes scabiei hominis • Treatment is permethrin 5%
Onychomycosis
Patient will be complaining of thickened and discolored toenails • Diagnosis is made by KOH preparation of nail scraping • Treatment is oral terbinafine • Comments: Serum aminotransferases should be monitored before starting treatment with terbinafine and during the treatment due to hepatotoxicity
Intertrigo
Patient will be obese • PE will show regions of erythema with peripheral scaling by skin folds • Most commonly caused by moisture, friction, and lack of ventilation
Acanthosis Nigricans
Patient will be obese or diabetic • PE will show thickened, velvety, darkly pigmented plaques on the neck or axillae • Comments: Screen for diabetes in those not yet diagnosed
Erythema Infectiosum (Fifth Disease)
Patient with a history of URI symptoms 3 - 4 days prior to rash • PE will show "slapped cheek" rash • Most commonly caused by parvovirus B19 • Comments: Aplastic crisis in sickle cell patients
Auto-Eczematization (ID Reaction)
Patient with a history of a recent fungal infection • Complaining of erythematous, papular rash over palms and fingers • Most commonly caused by delayed-type hypersensitivity reaction • Treatment is resolution of the primary infection
Atopic Dermatitis (Eczema)
Patient with a history of asthma or hay fever • Complaining of itchy, scaly rash on arms, often worse in the winter • PE will show thick, leathery, hyperpigmented areas on flexor surfaces • Treatment is topical corticosteroids, lubricating ointments
Lyme Disease
Patient with a history of being in the woods hiking or camping • Complaining of: . Stage I: erythema migrans (pathognomonic), viral-like syndrome (fever, fatigue, malaise, myalgia, headache) . Stage II: arthritis, myocarditis, bilateral Bell's palsy . Stage III: chronic arthritis, chronic encephalopathy • PE will show slightly raised red lesion with central clearing, erythema migrans (bulls-eye) rash • Most commonly caused by Borrelia burgdorferi carried by Ixodes tick • Treatment is doxycycline. Kids/pregnant - amoxicillin • Comments: Bilateral facial nerve palsy is virtually pathognomonic for Lyme disease
Which of the following is responsible for lice infestation of the head and scalp? Pediculus humanus capitis Pediculus humanus corporis Piedraia hortae Pthirus pubis
Pediculus humanus capitis Pediculus humanus capitis is the head louse responsible for pediculosis capitis, a condition caused by infestation of the scalp and hair. Also referred to as lice or lice infestation, pediculosis capitis is very common worldwide and most frequently affects children. The female louse lays eggs and attaches them to the base of the hair shaft. These eggs, or nits, are easily seen without magnification on physical exam. They are most commonly found behind the ears or at the posterior hairline. Nits take 8-9 days to hatch and 9-12 days to mature. The average lifespan of Pediculus humanus capitis is 30 days. Potential infestations often come to the attention of parents by school officials who send advisories home due to the contagious nature of lice. Parents and teachers often make the diagnosis of lice by seeing nits or live insects and the patient comes to the medical office for confirmation and treatment recommendations. Treatment is with topical pediculicides and a second treatment should be applied nine days after the first to ensure eradication. Pediculus humanus corporis (B) is the louse responsible for body lice. The body louse is larger than the head louse and does not live on the human body like Pediculus humanus capitis does. Instead it lives in clothing and crawls onto the human body to feed, then retreats. Piedraia hortae (C) is a fungal infection found in tropical climates that presents with white, brown, or black concretions on the hair shaft. It is on the differential along with lice for individuals with a travel history or who live in a tropical climate. Pthirus pubis (D) is the louse responsible for pubic lice or crabs. Infestations with pubic lice are generally sexually transmitted with severe infestations also affecting the axillae and facial hair.
A five-month-old girl presents to your ED with a worsening rash. Her regular doctor diagnosed her with eczema three days ago and the parents are trying topical emollients without effect. She is fussy and having difficulty sleeping. She has had no fevers. No one in the family has a history of asthma or atopy, however her five-year-old brother recently developed a similar rash. Your exam shows a well appearing and well nourished infant trying to scratch at the above rash. Of the following, which is the most appropriate topical treatment? Hydrocortisone 1% Mupirocin 2% Nystatin 100,000 units/gram Permethrin 5% cream
Permethrin 5% cream Scabies is a highly contagious infestation of the mite Sarcoptes scabiei causing a common polymorphic intensely pruritic rash in children. The classic presentation is that of linear burrows in the webs of fingers and toes but this is rarely seen. The lesions can be subtle, small erythematous nondescript papules, vesicles, or nodules that are often excoriated and may be tipped by a hemorrhagic crust. The distribution of the lesions varies depending on the patient's age, likely representing the different areas with which the mite has contact. In adults the lesions are often seen in the axillae, nipples, wrists, elbows, waist, and the groin. In infants and toddlers the distribution includes the head, neck, trunk, axillae, palms, soles, and ankles. Treatment is with permethrin 5% cream. The entire family and everyone who comes in contact with the infant should be treated simultaneously. Bed linens and other fomites should be cleaned or placed in a plastic bag for one week to interrupt the life cycle of the parasite. Permethrin dries the skin so the use of emollients is recommended after treatment. Scabies is often mistaken as eczema as the lesions can appear diffuse and eczematous, however hydrocortisone 1% (A) will only worsen the illness. Consider scabies in infants with recent widespread dermatosis without history of atopy. Impetigo can be treated with mupirocin 2% (B), however this infection typically has golden crusting not seen in our patient. Scabies can be confused with a fungal skin infection as it can have what appears to be satellite lesions, however the location is very unusual. Nystatin 100,000 units/gram (C) will not treat scabies.
A 24-year-old pregnant woman presents with a rash on her hands with itching that is worse at night. Physical exam shows excoriations over the web spaces of the hands with tiny burrows noted. Several members in her household have similar symptoms. Which of the following is the most appropriate treatment? Ivermectin Lindane 1% cream Permethrin 5% cream Sulfur in petroleum 5%
Permethrin 5% cream Permethrin 5% cream is a safe treatment for scabies during pregnancy. Less than 2% of an applied dose is absorbed systemically although residual effects of the drug remain for up to ten days after application. Scabies is caused by a mite infestation with Sarcoptes scabiei. These mites burrow under the skin of infested individuals leaving behind eggs and feces in the small epidermal tunnels that they create. These burrows are usually found in the web spaces between digits, intertriginous areas, and flexor creases. Symptoms include itching that is worse at night and a rash or itching that is worse in web spaces or intertriginous areas. History will often reveal others in the household with similar symptoms. Diagnosis of scabies is usually clinical, however, skin scrapings can reveal microscopic confirmation of adult mites or their eggs. Treatment includes permethrin 5% cream, lindane 1% cream (not available in the USA due to side effects), ivermectin, or crotamiton 10% cream. Permethrin 5% cream is safe during pregnancy and is applied over the entire body from the neck down and left on for eight hours. Afterwards, it is washed off entirely. Patients should also be instructed to launder all clothing and bed linen in hot water to prevent reinfestation. Ivermectin (A) should not be used during pregnancy and is not safe to use in children less than five years. It is a pregnancy category C drug and has shown to be teratogenic in studies using mice, rats, and rabbits. Lindane 1% cream (B) is a pregnancy category C drug and has been shown to be absorbed systemically in studies. There are concerns regarding neurotoxicity and hematotoxicity with its use making it an unsuitable choice for pregnant females or young children. Lindane is no longer available in the USA. Sulfur in petroleum 5% (D) is safe during pregnancy, however, the unpleasant odor and staining associated with this drug make it a less attractive choice for treatment.
How frequently should a patient with scabies use topical permethrin?
Permethrin will not kill unhatched mites so they must apply it twice, one to two weeks apart.
What medication is indicated for the treatment and prevention of head lice and scabies?
Permethrin.
A 35-year-old Caucasian woman presents with complaints of dark brown patches of skin on her upper lip and forehead. She first noticed the lesions last year while she was pregnant but it has persisted. What is the most important intervention to reduce worsening of this skin manifestation? Camouflaging agents Exfoliating agents Photoprotection Topical corticosteroids
Photoprotection Photoprotection is the most important intervention in reducing melasma. Melasma often presents during pregnancy and is characterized by light- to dark-brown patches of skin on the face. It is sometimes called "the mask of pregnancy". This is often secondary to hormonal changes and can occur in women taking oral contraceptives as well. The macular, hyperpigmented areas are either found on the forehead, cheeks, and upper lip area, the mandibular area, or in a malar distribution. Melasma can be divided into four clinical types: epidermal type, dermal type, mixed type, and indeterminate type. A Wood's lamp can be used to distinguish between the types. Camouflaging agents (A) can be used but are not useful in preventing worsening hyperpigmentation. Exfoliating agents (B) are often not helpful unless a patient practices photoprotection. Topical corticosteroids (D) have no place in treating melasma.
A 34-year-old man presents with alcohol intoxication and left hand pain. The patient states that he fell asleep on the sidewalk in the snow. Examination reveals a swollen, erythematous left hand with clear blisters as seen above. The patient has decreased sensation and decreased range of motion. What therapy is indicated? Place hand in warm (37°C/98.6°F - 39°C/102.2°F) circulating water Place hand under hot (45°C/113°F - 52°C/125.6°F) running water Warm hand with circulating hot air Wrap hand in warm blankets
Place hand in warm (37°C/98.6°F - 39°C/102.2°F) circulating water The patient presents with severe frostbite to the left hand from exposure and will require gentle, active rewarming of the extremity. During cold exposure, vasoconstriction occurs in an effort to conserve heat. As the temperature drops below 10°C, cutaneous sensation is compromised. With microvascular vasoconstriction, plasma begins to leak into the interstitial space. Ice crystals begin to form once the temperature approaches 0°C. Once crystals begin forming, intracellular osmolarity rises and cells begin to collapse and die. Blood flow begins to sludge followed by stasis and cessation of flow at the capillary level. Patients will often present with pain and decreased sensation (75%) but usually do not have frank frozen and insensate tissue. Frostbite, like burns, is classified into degrees of injury. First-degree frostbite is characterized by anesthesia and erythema. Second-degree frostbite will have superficial vesicles surrounded by edema. Third-degree frostbite produces hemorrhagic vesicles. Fourth-degree injuries extend deeper into osseous and muscle tissue. Optimal treatment should begin with removing all wet or cold clothing and assessing the patient for possible hypothermia. Any parts that are frozen should be submerged in warm circulating water (37°C - 39°C). Warming should not be initiated until it is certain that refreezing will not occur as this can cause more tissue damage. Passive rewarming with warm blankets (D) will not adequately increase the temperature to prevent further damage. Placing the frozen extremity in hot water (B) is often too painful for the patient to tolerate and may produce added heat injury. Application of dry heat (C) is also contraindicated as it leads to further damage and is poorly tolerated.
Sulfa Allergy
Popular FACTSSS: Probenecid, Furosemide, Acetazolamide, Celecoxib, Thiazides, Sulfonamides, Sulfasalazine, Sulfonylureas
Latent Tuberculosis
Positive PPD criteria: . 15 mm: no ↑ risk . 10 mm: high-risk, homeless, health-care workers, IVDU, foreign born . 5 mm: immunosuppressed, recent TB contact, abnormal CXR, steroid use • Rx: INH for 9 months
A 35-year-old man presents to the ED with a severe rash. A localized portion is seen in the image above. He states that he was cleaning away some brush from the woods behind his house a couple of days ago while only wearing shorts. On exam, you note similar lesions on his face, back, legs, arms, and chest. Which of the following is the most appropriate treatment? Cephalexin for 7 days Diphenhydramine as needed Prednisone taper over 21 days Prednisone taper over 7 days
Prednisone taper over 21 days The patient was exposed to poison ivy and developed allergic contact dermatitis. Contact dermatitis is an inflammatory reaction of the skin to a chemical, physical, or biologic agent. The inducing agent acts as an irritant or allergic sensitizer. Clothing, jewelry, soaps, cosmetics, plants, and medications contain allergens that commonly cause allergic contact dermatitis. The most common allergens include rubber compounds, plants of the Toxicodendron genus (poison ivy, poison oak, sumac), nickel (often found in jewelry), paraphenylenediamine (an ingredient in hair dyes and industrial chemicals), and ethylenediamine (a stabilizer in topical medications). Clinical presentation is variable with primary lesions being papules, vesicles, or bullae on an erythematous base. The distribution of the eruption depends on the specific allergen and may be localized, asymmetric linear, or unilateral. The classic lesion of poison ivy is a linear eruption that occurs from the person brushing against the poison ivy leaf. The rash usually appears 2-21 days after exposure and is associated with intense pruritus. Treatment for mild cases of contact dermatitis from poison ivy includes calamine lotion or a topical steroid and oral antihistamine. However, in moderate to severe cases, systemic steroids are indicated and should be continued for 2-3 weeks, with a gradual taper to prevent rebound of the disease. Systemic antihistamines can be taken concomitantly to help control the pruritus. Cephalexin (A) is a first-generation cephalosporin commonly used in the treatment of cellulitis. It is not used as a primary treatment in contact dermatitis but may be necessary if a secondary bacterial infection develops. Diphenhydramine (B), an antihistamine, is used as adjunctive therapy in severe cases and primary therapy for mild cases of allergic contact dermatitis. Prednisone (D) should be administered over 2-3 weeks rather than 7 days to avoid rebound of the disease. A 7-day course may treat the lesions temporarily, but often the rash returns.
What conditions commonly cause false positive PPD results?
Prior infection with nontuberculous mycobacteria and a history of a bacilli Calmette-Guerin (BCG) vaccination
What is the characteristic finding consistent indicative of corneal ocular involvement in zoster ophthalmicus?
Pseudodendrites (no terminal bulb).
A 23-year-old woman presents to the office with plaques on the extensor surfaces of her knees and elbows. She complains that the areas are itchy and they bleed after they are scratched. On exam, you also notice pitting of the fingernails. What is the most likely diagnosis? Erythema multiforme Lichen simplex chronicus Psoriasis Seborrheic Dermatitis
Psoriasis Psoriasis is an autoimmune condition with peak incidence in the early 20's. It occurs equally in males and females, and has a hereditary component. Stereotypical lesions are chronic, relapsing, thick silvery and scaly papules and plaques that overly an erythematous base. It is a chronic disorder but certain triggers such as infection, drugs, and trauma can cause flares. The Koebner phenomenon occurs when physical trauma elicits new growth of psoriatic patches. The most common areas affected are the extensor surfaces of the extremities (elbows and knees), lower back, palms and soles, scalp, gluteal cleft, and genital regions. Management is with glucorticoid topicals, UV-A therapy, and, in extreme cases, methotrexate and biologic medications. Erythema multiforme (EM) (A) is a cutaneous reaction to various triggers, most commonly herpes simplex virus. EM is also triggered by medications such as sulfonamides, phenytoin, and penicillin. Lesions are erythematous flat papules with a central vesicle, often described as target lesion. Lichen simplex chronicus (B) is a skin condition, which results from chronic rubbing or scratching. Skin appears thickened, skin markings are accentuated, and there is minimal scaling. This condition can occur anywhere on the body. Seborrheic dermatitis (D) can occur at any age, but is commonly seen in infancy and old age. Skin lesions are characterized by redness and scaling with a greasy appearance. It occurs on the scalp (in baby's referred to as cradle cap), but also occurs in the eyebrows, lashes, nasolabial folds, and body folds.
Tissue Adhesives
Rapidly applied • Equivalent cosmetic result • Resistance to bacterial growth • Avoid high tension lacerations • Use petroleum products to loosen bond
What other tick-borne disease presents as a faint maculopapular rash that begins on the wrists and ankles and spreads to the extremities and trunk?
Rocky Mountain spotted fever (Rickettsia rickettsi)
What kind of agar is used to grow dermatophytes in culture?
Sabouraud's agar.
A 19-year-old Hispanic man with a body mass index of 32 kg/m2 presents to your office with a complaint of darkened skin. Physical exam reveals thickened, velvety, darkly pigmented plaques on the back of his neck. Which of the following is the most appropriate next step in management? Referral to gastroenterology Screening for diabetes mellitus Skin biopsy Trial of topical retinoids
Screening for diabetes mellitus Acanthosis nigricans is a dermatologic condition characterized by darkened plaques, most commonly in the intertriginous sites such as the axillae or neck. A significant percentage of patients who are obese or have diabetes have this condition, and the presence of acanthosis nigricans in children is a risk factor for the development of diabetes. Ethnic differences are seen with a higher percentage of African-American and Hispanic patients having the condition than non-Hispanic whites or Asians. Rarely, acanthosis nigricans can develop as a result of malignancy. Diagnosis is by clinical examination and assessment for other disorders, including screening for diabetes mellitus, is an important next step in the evaluation of patients with this condition. Acanthosis nigricans is a benign condition and treatment is often based on cosmetic concerns. Treatment of the underlying condition is the preferred method of management. Older, non-obese adults with new onset acanthosis nigricans should be referred to gastroenterology (A) because of the risk of malignancy, most often due to tumors in the gastrointestinal tract. This is done after ruling out more common metabolic conditions. Skin biopsy (C) may be done if the clinical diagnosis is difficult to determine. There is no definitive treatment for acanthosis nigricans outside of treating the underlying condition. Topical medications including retinoids (D) have been effective in some cases of acanthosis nigricans, but are not the first step in management.
What is the most common complication of atopic dermatitis?
Secondary bacterial infections due to frequent disruption of the skin barrier.
Psoriasis
Silvery scales • Extensor surfaces • Nail pitting • Auspitz sign: scale removal produces blood droplets • Arthritis
What are first-line treatments for melasma?
Skin-lightening agents and retinoids.
How is scabies transmitted?
Skin-to-skin contact.
Human papillomavirus is a risk factor for the development of what skin malignancy?
Squamous cell carcinoma.
What is the recommended treatment for dyshidrotic ecematous dermatitis?
Steroids can be administered topically, intralesionally or systemically depending on the severity. PUVA (psoralen and UVA light) may also be beneficial.
A 23-year-old woman complains of headaches and arthralgias for the past 3 days. She presents to urgent care because of a painful rash on her face that has progressed into multiple vesicles. This rash is characterized by erythematous macules with dark purple centers with multiple vesicles in different stages. On physical exam there is also mucosal edema within the oral cavity. She recently completed an antibiotic course for a urinary tract infection. Which of the following is the most likely diagnosis? Erythema multiforme minor Staphylcoccal scalded skin syndrome Stevens-Johnson syndrome Toxic epidermal necrolysis
Stevens-Johnson syndrome Stevens-Johnson syndrome is an immune-complex-mediated hypersensitivity reaction that typically involves the skin and the mucous membranes. After one to three weeks of exposure to the causative agent, a prodrome of fever, malaise, headache, cough, and conjunctivitis develops. Skin lesions appear one to three days after this prodrome. The lesions initially appear as erythematous macules with dark purpuric centers, then form atypical target lesions with central dusky purpura or a central bulla, with surrounding macular erythema. Mucosal involvement occurs in almost all affected patients. By definition, Stevens-Johnson syndrome affects less than 10% of the body surface area. Approximately 50% of cases of Stevens-Johnson syndrome are drug induced. Common causative agents include sulfa drugs, antiepileptic drugs, antibiotics, and nonsteroidal anti-inflammatory drugs. Steven-Johnson syndrome is a clinical diagnosis. The causative drug and any unnecessary medications should be discontinued. Aggressive management should include fluid resuscitation, nutritional supplementation, and wound care. Antibiotics should be used only if there is evidence of infection. Intravenous immune globulin has been incorporated into some treatment protocols. Complications include hypotension, renal failure, corneal ulcerations, anterior uveitis, erosive vulvovaginitis, respiratory failure, seizures, and coma. Staphylococcal scalded skin syndrome (B) most commonly occurs in infants and in young children, faint, orange-red, macular erythema with cutaneous tenderness. Periorificial and flexural accentuation may be observed. Characteristic tissue paper-like wrinkling of the epidermis is followed by the appearance of large, flaccid bullae in the axillae, in the groin, and around the body orifices. Toxic epidermal necrolysis (D) is a potentially life-threatening dermatologic disorder characterized by widespread erythema, necrosis, and bullous detachment of the epidermis and mucous membranes, resulting in exfoliation. By definition, toxic epidermal necrolysis affects more than 30 percent of the body surface area. Erythema multiforme minor (A) represents a localized eruption of the skin with minimal or no mucosal involvement. The papules evolve into pathognomonic target lesions that appear within a 72-hour period and begin on the extremities. Lesions remain in a fixed location for at least 7 days and then begin to heal.
What common infection is known to trigger eruptive, inflammatory psoriasis?
Streptococcal Pharyngitis.
What is the most serious long-term complication of measles infection?
Subacute sclerosing panencephalitis.
What has a greater risk of infection, cutaneous or subcutaneous sutures?
Subcutaneous.
Dyshidrotic eczema flares are more frequent in which season?
Summer.
Why is trimethoprim-sulfamethoxazole (TMP-SMX) a poor choice as a single agent to treat cellulitis?
TMP-SMX is active against most methicillin resistant Staphylococcus aureus but has poor coverage against group A streptococci; an organism often implicated in cellulitis.
A 14-year-old boy developed an erythematous, papular rash over his palms and fingers. A complete physical examination reveals scaling and cracking in the interdigital webs of his feet. No other areas are affected. What is the most appropriate treatment? Intramuscular benzathine penicillin G Permethrin cream applied from the neck down Supportive care for hand-foot-and-mouth disease Terbinafine cream applied twice daily to feet
Terbinafine cream applied twice daily to feet The above patient presents with auto-eczematization, also known as an id reaction, which is thought to be due to a delayed-type hypersensitivity reaction to a fungal infection. In this case, it is secondary to his tinea pedis. Tinea pedis, also known as athlete's foot, is the most common dermatophytic fungal infection. Affected individuals develop vesicles or papules over the soles and interdigital webs of the feet, followed by scaling and cracking. It most commonly results in sterile vesicles over the palms and soles. The id reaction resolves with treatment of the primary infection. The treatment of choice for tinea pedis is a topical antifungal, such as terbinafine or clotrimazole, applied to the affected area for four weeks. The correct answer is terbinafine applied twice daily to feet. Intramuscular benzathine penicillin G (A) is the treatment of choice for syphilis. Although the rash of secondary syphilis typically includes the palms and soles, it does not include cracking and scaling of the interdigital webs. The lesions caused by secondary syphilis vary widely in appearance and may be maculopapular, pustular, or annular. Lesions are typically erythematous or brown in color. Although less likely than an id reaction, syphilis should be considered in any adolescent with a rash of the palms and soles. Permethrin cream applied from the neck down (B) is the treatment of choice for scabies. While scabies does affect the interdigital webs of the hands, it does not cause scaling or cracking of the interdigital webs of the feet unless significantly excoriated. Moreover, scabies is intensely pruritic and spreads rapidly over the skin of the affected individual and also within families. Supportive care for hand-foot-and-mouth disease (C) is incorrect. While the child's hand lesions may resemble those of hand-foot-and-mouth disease, he lacks characteristic oral ulcers, rash on feet, or systemic symptoms of infection. Notably, if the patient did have hand-foot-and-mouth disease, supportive care, including pain control, would be the treatment of choice.
What is a common complication within 24 hours of initiation of antibiotic treatment of Lyme disease?
The Jarisch-Herxheimer reaction.
What is the recommended method of burn assessment in children?
The Lund-Browder chart is the recommended method because it takes into account the relative percentage of body surface area affected by growth.
Which of the following statements regarding allergic drug reactions is true? Celecoxib often causes a reaction in patients with a sulfonamide allergy Cross-reactivity between penicillins and first generation cephalosporins is 50% Patients with a history of anaphylaxis to penicillin can never be given penicillin The onset of serum sickness generally occurs within one to two weeks
The onset of serum sickness generally occurs within one to two weeks Serum sickness (D) is an immune-complex mediated reaction characterized by malaise, joint pain, urticaria, fever, adenopathy, and hepatosplenomegaly. Symptoms usually begin one to two weeks after drug exposure and may take several weeks to resolve. Treatment is generally supportive with corticosteroids administered for more severe cases. Celecoxib contains a sulfonamide moiety (A), but recent data shows that the cross-reactivity between antibiotic sulfonamides and non-antibiotics sulfonamides may not occur at all or at the very least this potential is extremely low. Specifically, the mechanisms of cross-reactivity causing anaphylaxis are very unlikely to occur when using a non-antibiotic sulfonamide. The true cross-reactivity rate is unclear. The rate of cross-reactivity of penicillins and first generation cephalosporins is approximately 1%-7%, not 50% (B). Third and fourth generation cephalosporins have a much lower cross reactivity than first generation, closer to 1%. Many patients with a penicillin allergy can tolerate second- and third-generation cephalosporins without issue. However, patients with life-threatening reactions to penicillin should not receive cephalosporins in the ED unless absolutely necessary. Patients with a history of penicillin anaphylaxis should not be given penicillin or cephalosporins, however, there are a few selected indications where desensitization (C) should be performed. An example is the pregnant patient with syphilis and a penicillin allergy. Careful desensitization should occur in the ICU with increasing doses of the medication.
Where are the most commonly involved sites of HPV infection in men?
The penis and scrotum.
What is black dot Tinea capitis?
This refers to an infection that causes the hair to fracture, leaving the infected dark stubs visible in the infected regions.
An 11-year-old girl presents with an itchy lesion that has been present on her proximal left arm for one week. It began as a small, scaly red patch and has spread outward to form an erythematous circle with a whitish center. The bordering edges appear slightly more raised than the rest of the circle. What is the most likely diagnosis? Granuloma annulare Nummular eczema Pityriasis rosea Tinea corporis
Tinea corporis The above child has classic findings of tinea corporis, a dermatophyte infection that occurs in both immunocompromised and immunocompetent hosts. The lesion begins as an erythematous, raised, scaly plaque that enlarges centrifugally. The borders may become more raised than the rest of the lesion, while the center begins to clear. As multiple lesions often occur in the same individual, examination of the entire body surface area is necessary. The diagnosis is often clinically apparent, but in uncertain cases, KOH preparation of skin scrapings may be used to visualize hyphae. First-line treatment of tinea corporis in an immunocompetent child is a topical azole antifungal. Granuloma annulare (A) is a benign inflammatory lesion of unknown etiology. Similar to tinea corporis, granuloma annulare has an annular shape and may be erythematous with central clearing. However, it lacks the classic scaling of tinea corporis. Nummular eczema (B) is a relatively uncommon form of eczema that causes multiple coin-shaped lesions over the extremities and, less commonly, the trunk. Scaling and central clearing may occur but not until later in the course of each lesion. Pityriasis rosea (C) is a common childhood exanthem, beginning with a characteristic herald patch, an annular plaque that is located on the upper back, neck, or chest. New, oval-shaped, erythematous papules then develop in a classic "Christmas tree" distribution over the the cleavage lines of the trunk. In children, the scalp, face, and distal extremities may also be involved. In contrast to tinea corporis, the multiple lesions arrive and evolve in crops and self-resolve in four to six weeks. The etiology of pityriasis rosea is unknown.
A 17-year-old boy is in your clinic because of itchiness on both feet. He noted lesions on his feet for about two weeks that are accompanied by pruritus and pain. He denies any history of allergies or trauma. On physical examination, vital signs are normal. On skin examination, there are pruritic, erythematous erosions and scales in the interdigital areas of the toes of both feet. There are also some interdigital fissures seen. Which of the following is the most likely diagnosis? Erythrasma Interdigital Candida infection Palmoplantar psoriasis Tinea pedis
Tinea pedis The boy has findings that are consistent with tinea pedis. Tinea pedis (also known as athlete's foot) is the most common dermatophyte infection. Tinea pedis may manifest as an interdigital, hyperkeratotic, or vesiculobullous eruption. Interdigital tinea pedis is most common. Tinea pedis usually occurs in adults and adolescents (particularly young men). Common causes are Trichophyton rubrum, Trichophyton interdigitale and Epidermophyton floccosum. Infection is usually acquired by means of direct contact with the causative organism, as may occur by walking barefoot in locker rooms or swimming pool facilities. The boy has interdigital tinea pedis that manifests as pruritic, erythematous erosions or scales between the toes, especially in the third and fourth digital interspaces. Associated interdigital fissures may cause pain. The diagnosis is confirmed with the detection of segmented hyphae in skin scrapings from an affected area with a potassium hydroxide (KOH) preparation. Erythrasma (A) is the most common cause of interdigital bacterial infection on the feet. It exhibits scaling and maceration of the web spaces, usually between the fourth and fifth toes. Interdigital Candida infection (B) exhibits erythematous, macerated plaques and erosions with fine peripheral scaling and erythematous satellite papules and pustules. The pustules are easily ruptured, leaving an erythematous base with a surrounding collarette of epidermis. The plaques are often pruritic and occasionally are painful if significant skin breakdown accompanies the infection. Palmoplantar psoriasis (C) presents with clusters of white to yellow-brown sterile pustules, erythema, and scale on the palms and soles and appears strongly associated with smoking.
What term solely refers to dermatophyte nail infections?
Tinea unguium.
Which fungal infection may present with areas of pink or white macules, commonly on the upper torso, that may be confused with vitiligo in patients of dark complexions?
Tinea versicolor may present as pale macules that do not tan, commonly on the upper trunk.
A 28-year-old woman with a history of pernicious anemia presents with significant depigmentation of the skin on her cheeks, hands and forearms. She has no history of damaging events to the skin or prior dermatologic disorders. Which of the following interventions would be the best initial step in managing this patient? Local liquid nitrogen application Narrowband UVB therapy Oral PUVA therapy Topical tacrolimus
Topical tacrolimus Topical tacrolimus 0.1%, applied twice daily, is the first-line therapy for patients with vitiligo affecting less than 20% of the patient's body. Vitiligo is suspected in patients with any hypopigmentation in the color of their skin compared to the surrounding skin areas. The underlying cause of vitiligo is the destruction of melanocytes, or pigment cells. Often, there is an autoimmune component to this destruction and it may be seen in conjunction with diseases such as pernicious anemia, autoimmune thyroid disease, Addison disease, and type 1 diabetes. In the absence of an underlying autoimmune condition, other causes of hypopigmentation may be responsible. A history of using intralesional corticosteroids or liquid nitrogen over the affected skin may cause temporary loss of pigmentation, particularly in patients with a naturally olive or dark complexion. True vitiligo should also be distinguished from post-inflammatory hypopigmentation, as is seen in patients with a history of inflammatory skin conditions. A Wood's lamp can be used to accentuate vitiligo, as the hypopigmentation following an inflammatory condition will not enhance readily. Treating vitiligo is often a long process of trial-and-error. In patients with less than 20% involvement, topical tacrolimus can be recommended. Super-potent topical steroid creams may be used as well, but with the additional risk of skin atrophy in prolonged courses. Patients with 20-25% involvement may benefit more from narrowband UVB or oral PUVA therapy. However, patients should be counseled that there is a risk of severe phototoxic reactions with PUVA therapy. Generally, the fingertips and genitals are most difficult to treat, while the face and chest can be expected to respond well. Years of treatment are usually needed. Importantly, vitiligo increases a patient's risk of developing skin cancers, and more thorough skin screening methods should be considered. Local liquid nitrogen application (A) is generally not recommended to treat the hypopigmentation seen in vitiligo. In fact, patients with naturally darker complexions are at risk of developing permanent hypopigmentation after using liquid nitrogen to treat other dermatologic conditions. Narrowband UVB therapy (B) and oral PUVA therapy (C) are recommended treatments for vitiligo. However, these are not considered first-line treatments in patients with less than 20% skin involvement, as they carry an increased risk of phototoxic reactions.
A 13-year-old boy presents with fever, arthralgias, lympadenopathy and pruritus 7 days after starting phenytoin for a newly diagnosed seizure disorder. What type of hypersensitivity reaction explains the patients symptoms? Type I Type II Type III Type IV
Type III The patient is suffering from a serum sickness-like reaction caused by phenytoin, which is a type III hypersensitivity reaction. Type III hypersensitivity reactions are immune complex-mediated reactions. Antibodies bind to antigens to form immune complexes. These are deposited on vessel walls leading to local inflammation and eventually tissue injury. Type III reactions are seen in systemic lupus erythematosus and serum sickness. Serum sickness presents with rash, fever, polyarthralgias and polyarthritis usually begins 1-2 weeks after exposure to a causative agent. Type I (A) reactions are immediate hypersensitivity where binding of antigens to IgE on mast cells and basophils leads to deranulation of mediators. Type II (B) reactions involve cell lysis resulting from antibody binding to membrane-bound antigens. Type IV (D) reactions or cell-mediated delayed hypersensitivity requires sensitized lymphocytes to start the inflammatory reactions. Type IV reactions are seen in contact dermatitis, Stevens-Johnson syndrome and toxic epidermal necrolysis.
Serum Sickness
Type III hypersensitivity • Onset usually 7-21 days after exposure or 12-36 hours after a reexposure • Flulike sx, rash
Which of the following sutures is absorbable? Nylon (Polyamide) Prolene (Polypropylene) Silk (Silk) Vicryl (Polyglactin)
Vicryl (Polyglactin) Absorbable sutures are degraded and lose strength in less than 60 days. Absorbable sutures are generally used for subcutaneous and mucosal closures. Their highly reactive nature allows them to be broken down and absorbed over weeks. Vicryl is an absorbable braided polymer of lactide and glycolide with low reactivity and good strength making it suitable for subcutaneous and mucous membrane use. Prolene (B), Nylon (A), and Silk (C) are nonabsorbable.
A 19-year-old is brought to the ED via ambulance after he was found sleeping outside in a snowstorm. The ambient temperature is 25°F. His vital signs are T 36.0°C, BP 125/70 mm Hg, RR 14, and HR 75. He complains of stinging and burning pain in all ten fingers. On exam, you note edema, erythema, and multiple blisters beginning to form over the fingers and hands. Which of the following is the most appropriate next step in management? Administer intravenous antibiotics Debridement of blisters Dry rewarming at 98.6°F (37°C) Wet rewarming at 98.6°F (37°C)
Wet rewarming at 98.6°F (37°C) This patient has evidence of moderately severe (second-degree) frostbite. Rapid rewarming is the cornerstone of frostbite therapy and should be initiated as soon as possible. The injured extremity should be placed in gently circulating water (ideally a whirlpool) at a temperature of 98.6°F to 102.2°F (37°C to 39°C) for approximately 15 to 30 minutes until the distal extremity is pliable and erythematous. The role of prophylactic antibiotics (A) is unclear. Some protocols recommend intravenous penicillin; others recommend topical bacitracin. Nonetheless, the most important initial step is to begin rapid rewarming. The blisters (B) in second-degree frostbite are rich in destructive thromboxane and prostaglandins. Although removal of these blisters results in removal of these destructive substances, not all experts agree that debridement is necessary. Hemorrhagic blisters should never be debrided because this can result in tissue desiccation. However, treatment of the blisters is secondary; the priority is rapid rewarming. Dry rewarming (C) is not recommended and is associated with induction of further injury.
What is the phenomena of "core temperature after-drop" in frostbite management?
When large areas are rewarmed, vasoconstriction is relieved and cold, hyperkalemic and acidic blood returns to the central circulation.
What is the definition of Stevens-Johnson/toxic epidermis necrolysis?
When the body surface area involved in 15-30%.
Frostbite
• Do not rewarm if possibility of refreezing • Rx: address hypothermia first, rapid rewarming of affected part in 37-39°C water
Contact Dermatitis
• Patient will complain of a rash • PE will show erythematous, scaly plaques, vesicles, and bullae • Diagnosis is made by patch test • Most commonly caused by nickel, poison ivy, soaps, and clothing • Comments: Cell-mediated reaction type IV